LSAT and Law School Admissions Forum

Get expert LSAT preparation and law school admissions advice from PowerScore Test Preparation.

 Administrator
PowerScore Staff
  • PowerScore Staff
  • Posts: 8917
  • Joined: Feb 02, 2011
|
#23508
Complete Question Explanation

Weaken. The correct answer choice is (A)

In this stimulus, the author makes what seems to be a reasonable argument. Since fines are higher than the cost of taking required precautions, businesses will now install adequate safeguards. One possible problem with this argument is that we know little about the chances of having to pay those fines.

Since this is a weaken question, we might look for the answer choice that shows the fines to be ineffective.

Answer choice (A): This is the correct answer choice. If businesses generally underestimate the risk of accidents, then even if the risk is high that may not be sufficient to induce installment of safeguards.

Answer choice (B): The prioritization of long- versus short-term profits is irrelevant to the authors argument about the effectiveness of the fines.

Answer choice (C): This is consistent with the author's perspective, and thus does not weaken the argument.

Answer choice (D): How businesses record fines for accounting purposes plays no role in the argument that the cost of the fines would induce installation of safeguards.

Answer choice (E): Like the other answer choices above, this one discusses an irrelevant consideration: in this case, it is that of business promotional strategy. Since this would in no way weaken the authors argument about fines and safeguards, this answer choice is incorrect.
 biskam
  • Posts: 124
  • Joined: Aug 18, 2017
|
#40107
I immediately crossed out A because I thought it strengthened the conclusion that "those that might have such accidents will now install adequate environmental safeguards." My thinking was if the risk of future accidents are actually greater than what they they think they are, it makes it all more important to install those safeguards. My prephase was... look for something that says installing safeguards isn't necessary and I thought this did just the opposite.

Thanks in advance!
 nicholaspavic
PowerScore Staff
  • PowerScore Staff
  • Posts: 271
  • Joined: Jun 12, 2017
|
#40138
biskam wrote: My thinking was if the risk of future accidents are actually greater than what they they think they are, it makes it all more important to install those safeguards. My prephase was... look for something that says installing safeguards isn't necessary and I thought this did just the opposite.
Hi biskam,

Weakening questions are tricky because they demand nuance. It is very rare when a correct answer is the 100% opposite of what the conclusion is in a stimulus. So prephrasing the dead opposite of the conclusion is not going to be an effective way of attacking a conclusion. In other words, your thinking about what Answer (A) describes is correct. If installing safeguards isn't a necessity, that is precisely why it weakens the stimulus. It negates it. It's not the polar opposite, but it negates and weakens it. That is the pattern to try with your prephrase and it is why Answer (A) is correct.

Thanks for the great question! :-D
 biskam
  • Posts: 124
  • Joined: Aug 18, 2017
|
#40147
nicholaspavic wrote:
biskam wrote: My thinking was if the risk of future accidents are actually greater than what they they think they are, it makes it all more important to install those safeguards. My prephase was... look for something that says installing safeguards isn't necessary and I thought this did just the opposite.
Hi biskam,

Weakening questions are tricky because they demand nuance. It is very rare when a correct answer is the 100% opposite of what the conclusion is in a stimulus. So prephrasing the dead opposite of the conclusion is not going to be an effective way of attacking a conclusion. In other words, your thinking about what Answer (A) describes is correct. If installing safeguards isn't a necessity, that is precisely why it weakens the stimulus. It negates it. It's not the polar opposite, but it negates and weakens it. That is the pattern to try with your prephrase and it is why Answer (A) is correct.

Thanks for the great question! :-D
Hi Nicholas,

Thanks for your response but I'm still struggling to understand how it weakens it... if the risk might be higher than it actually it is, doesn't it make it even more necessary to install the safeguards? I'm not seeing any negation, only solidification...

Thanks again!
User avatar
 Stephanie Oswalt
PowerScore Staff
  • PowerScore Staff
  • Posts: 811
  • Joined: Jan 11, 2016
|
#40176
Hey biskam!

I believe one of our instructors let us know they answered your question via the homework hotline, but please let us know if you have additional questions! :-D

Thanks!
 karen4300
  • Posts: 9
  • Joined: Jun 28, 2018
|
#47156
Hi! I'm just a bit confused with A). Does it weaken the conclusion because if businesses underestimate the risk of future accidents, then they will not adequately install enough safeguards?
 Adam Tyson
PowerScore Staff
  • PowerScore Staff
  • Posts: 5153
  • Joined: Apr 14, 2011
|
#47260
Correct, karen4300! The author here assumes that business KNOW that it would be more cost-effective to install safeguards than to risk accidents, but if they underestimate the risks they may not accurately make the true cost/benefit calculation. If they think the odds of an accident are much slimmer than they really are, they may be more willing to take that risk rather than pay the costs now.

Well done!
 andriana.caban
  • Posts: 142
  • Joined: Jun 23, 2017
|
#63397
Hi,

After reading the premise, I prephrased an answer choice that sounded much like answer choice (D). Since the conclusion is "[business] that might have accidents will now install adequate environmental safeguards" I thought, well what if the business decides to take the risk and treat a fine as an ordinary business expense if they even get a fine.

Can you help me see why (D) is a strengthen?
 Malila Robinson
PowerScore Staff
  • PowerScore Staff
  • Posts: 296
  • Joined: Feb 01, 2018
|
#63409
Hi andriana.caban,
I don't think that Answer D strengthens the argument. Instead, it is unclear what it does for the argument. Even if it is considered to be an ordinary business expense, if there is a less expensive way it would still make sense for businesses to chose that route in order to maximize profits. As such, it is irrelevant to the argument.
Hope that helps!
-Malila
 thecmancan
  • Posts: 11
  • Joined: May 02, 2019
|
#68187
I picked D also.

This answer choice baited my wrong selection because I made extra assumptions to make it stand up.

The argument in the stimulus is Because businesses value profits, they will install those safeguards to avoid the expensive fines.

However, I thought D weakened and made the safeguards less necessary because this:

If the businesses put the fines as a business expense, the cost are passed along to the customer in rising prices and what not.

Of course, bringing in real world knowledge of this actually happening is not a good idea for LSAT.

I did this because I dismissed A very quickly. I'm still very miffed on this AC. Will think on it more and post again if a light bulb happen to go off soon.

Get the most out of your LSAT Prep Plus subscription.

Analyze and track your performance with our Testing and Analytics Package.